ChaseDream
搜索
返回列表 发新帖
00:00:00

A major network news organization experienced a drop in viewership in the week following the airing of a controversial report on the economy. The network also received a very large number of complaints regarding the report. The network, however, maintains that negative reactions to the report had nothing to do with its loss of viewers.

Which of the following, if true, most strongly supports the network's position?

正确答案: A

更多相关帖子

524

帖子

15

好友

4712

积分

ChaseDream

注册时间
2003-03-17
精华
8
解析
查看: 2717|回复: 5
打印 上一主题 下一主题

OG13 19题的疑问

[复制链接]
跳转到指定楼层
楼主
发表于 2014-5-28 14:55:20 | 只看该作者 回帖奖励 |倒序浏览 |阅读模式
19. A major network news organization experienced a drop in viewership in the week following the airing of a controversial report on the economy. The network also received a very large number of complaints regarding the report. The network, however, maintains that negative reactions to the report had nothing to do with its loss of viewers.

Which of the following, if true, most strongly supports the network’s position?
(A) The other major network news organizations reported similar reductions in viewership during the same week.
(B) The viewers who registered complaints with the network were regular viewers of the news organization’s programs.
(C) Major network news organizations publicly attribute drops in viewership to their own reports only when they receive complaints about those reports.
(D) This was not the first time that this network news organization has aired a controversial report on the economy that has inspired viewers to complain to the network.
(E) Most network news viewers rely on network news broadcasts as their primary source of information regarding the economy.
因果无疑。尾句是结论。
答案是A.

我的疑问是:
根据Helr的讲义,果因需要:
1、CQ1他因排除(同果异因):还有别的原因,所以这个结论不是唯一的(weaken);没有别的原因可能会导致这个结果(Strengthen)
2、CQ2否定因果关系,前提和结论根本没有因果关系;

这里A选项只说了The other major network news organizations也有观众下降,但是并没有说The other major network news organizations是不是也报道了a controversial report on the economy啊!也就是说The other major network news organizations可能也报道了a controversial report on the economy,然后导致了reductions in viewership during the same week。按照常识一个新闻一般不会只在一个新闻媒体报道,很有可能所有的新闻媒体都播报了,而且又是the same week,这更增大了report导致收视率下降的这种可能性啊(相当于增大了sample base的数量:由个体行为扩展到整体行为)!这不等于削弱了结论(结论是没关系,削弱就是说有关系)?

如果A说的是The other major network news organizations对这个report与上文中的network news organization观点刚好相反;或者题干中说这个network news organization是唯一一个持这种观点的新闻网,那么A选项就是完美的答案。但是没这么说啊!
所以我可不可以认为A选项正确的前提是:我们需要假设The other major network news organizations没有报道a controversial report on the economy。

同样,请看D选项:这个新闻网之前也播过类似观点的report导致了观众的投诉。对收视率的影响有两中可能性:
1、下降了。以前播过收视率也下降,这次播收录率也下降,增大了播放这种 controversial report on the economy导致收视率的可能性(相当于增大了sample base的数量:由一次行为变为多次行为)。削弱了结论。
2、没下降。以前播收视率不下降,这次播收录率下降了,说明收视率降不降跟播不播controversial report on the economy没关系啊。支持了结论。
如果我们做一个跟A选项类似的假设:这个新闻网之前也播过类似观点的report导致了观众的投诉,但没有导致收视率的下降。这样的话,D也可能是正确答案啊!

我的疑惑就是既然A成立需要做一个假设(可能播可能没播,假设没播),那么为什么D不可以做一个类似的假设(可能降可能没降,假设没降)呢?
这不是A和D都有可能是正确选项吗?

请解疑。
收藏收藏1 收藏收藏1
沙发
发表于 2014-6-12 16:44:37 | 只看该作者
看过一些讨论 open to discuss 啊
供参考

确实A 和 D 都有可以讨论的余地
但是,题目问的是 MOST strongly support
因此一定是可以选一个更佳的出来

D
如果以前播放争议report, viewship 下降了,确实一定程度削弱了原文的论证
如果以前播放争议report, viewship 没下降,最多只能说是“不削弱”原文论证,但是也谈不上是“支持”
为什么呢?它不能论证 争议report 一定不导致 viewship 下降
如果这次的争议比以往更大所以这次viewship会下降而之前那次没下降?
我们不知道两次之间到底有什么因素的差别
某instructor:
http://gmatclub.com/forum/a-major-network-news-organization-experienced-a-drop-in-136966-20.html

Remember, past doesn't predict the future accurately and the future doesn't need to mirror the past. Hence, more often than not, past events will not provide much support to the future events. But we only have the past as reference to what will happen in the future so sometimes we base our hypothesis on the past.

我觉得是make sense 的,过去的因素和现在千差万别,只能提供很微弱的参考

相比 A 提出了 POSSIBLE explanation
even though it is not something 100% confirmed, it surely raises some supports.

再简单一点来看,
当我们推论 A 导致了 B
不管支持还是削弱 都需要在选项里面看到 A 和 B 的关系
D 里面 只提到了 有争议的报告 没有提到 viewship 这个结果
A 里面至少是有暗示的 原因 和 以及viewship 这个结果 因此要优于D

最后一点
对于 air a report 你的理解值得探讨一下
这个新闻台不是 “报道了a controversial report" (i quote)
它是发布了自己的带有争议的一个报道
并不是在提供一个客观的新闻事件
比方说 一个新闻是”失业率一直下降,本台认为奥巴马应该下台“
另一个新闻是”北部产生森林大火“
我个人认为 题目里说的report应该是前者
也就是说 不仅仅是报道一个新闻 而是提供了一个带有original 观点的报道
因此让A选项other possible exlanation 更有可能

the bottom line is:
选最优的,比较永远是王道~

板凳
 楼主| 发表于 2014-6-13 12:17:03 | 只看该作者
brfmzbjz 发表于 2014-6-12 16:44
看过一些讨论 open to discuss 啊
供参考

根据我的理解“支持和削弱”都不是绝对的,也就是说只是“相对的”支持和削弱,即使选项起了支持或者削弱作用也并不保证结论一定正确或者错误,只是增大了或者减少了结论成立的可能性而已。具体例子就是“乌鸦成群飞那道题”。
你说的:
如果以前播放争议report, viewship 没下降,最多只能说是“不削弱”原文论证,
我觉得:收视率降不降跟播不播controversial report on the economy没关系的可能性增大了,等于支持了结论:maintains that negative reactions to the report had nothing to do with its loss of viewers。
请指点。
地板
发表于 2014-6-13 12:34:37 | 只看该作者
alzn2765 发表于 2014-6-13 12:17
根据我的理解“支持和削弱”都不是绝对的,也就是说只是“相对的”支持和削弱,即使选项起了支持或者削弱 ...

没错
viewership 下降跟 controversial report 没关系的可能性增大了
就算不是我说的“不削弱”,也仅仅能是“很微弱很微弱的支持”,因为它毕竟是从反面说”A与B无关的可能性增加“

on the other hand,
A 直接从正面告诉我们 other possible factor accounts for the result. 相当于告诉我们”其实是C导致的B哦“,是不是支持得更有力一些?

D在说,凶手不那么可能是我!
A在说,凶手很有可能是小明!

确实是很纠结的题,但是关键得让自己想法化繁为简一些,这不是一个什么很大的逻辑问题,是我们想太多了而已~
5#
 楼主| 发表于 2014-6-16 11:50:09 | 只看该作者
嗯 明白了
问一下一般要达到多少的准确率才算CR过关了?
6#
发表于 2014-6-16 13:51:27 | 只看该作者
alzn2765 发表于 2014-6-16 11:50
嗯 明白了
问一下一般要达到多少的准确率才算CR过关了?

我了解的是 GMAT按题目难度而不是正确率给分
所以其实很难说做对多少个就拿多少分
不过一些帖子说 凭经验来看 模考错误要小于10个才能上700
语法比较难 按错5个来算的话
逻辑只能容错2-3个吧
您需要登录后才可以回帖 登录 | 立即注册

Mark一下! 看一下! 顶楼主! 感谢分享! 快速回复:

手机版|ChaseDream|GMT+8, 2024-4-28 07:19
京公网安备11010202008513号 京ICP证101109号 京ICP备12012021号

ChaseDream 论坛

© 2003-2023 ChaseDream.com. All Rights Reserved.

返回顶部